23
$\begingroup$

I am teaching Mathematical analysis. A student asked this question. I think this is a good question, but don't know the answer.

$\endgroup$

3 Answers 3

11
$\begingroup$

Here's an elementary construction that may be more digestable by your student.

  1. By definition of left continuity, for every point $x$ and every $\epsilon > 0$ there exists $\delta > 0$ such that if $y\in (x-\delta,x)$, $|f(y)-f(x)| < \epsilon$. We can upgrade this by the triangle inequality to for every $y_1,y_2\in (x-\delta,x)$, $|f(y_1) - f(y_2)| < 2 \epsilon$.

  2. Now we construct a sequence of points $x_i$ as follows. Let $\epsilon_i = 2^{-i}$ be fixed. Let $x_1$ be arbitrary, and $\delta_0 = 1$. Given $x_k$, we define $\tilde{\delta}_k$ be that which is given by step 1. And we define $$\delta_k = \min(\tilde{\delta}_k, \frac13 \delta_{k-1})$$ And we let $x_{k+1} = x_k - \frac13 \delta_k$.

  3. Let $z = \lim x_k$. Note that $z < x_k$ for all $k$ and $\frac13 \delta_k < x_k - z < \frac23 \delta_k$, so by construction we have that whenever $y \in (z - \frac13 \delta_k, z+\frac13\delta_k)$ we have $|f(x) - f(y)| < 2^{1-k}$. This shows continuity of $f$ at $x$.

$\endgroup$
14
$\begingroup$

Let $\omega(f,A):= \sup \{|f(x)-f(y)| : x,y \in A\}$ be the oscillation of $f$ on $A$, and let $O_n$ be the union of all open sets $A$ s.t. $\omega(f,A) < 1/n$. Then $O_n$ is open and its complement is countable because every uncountable closed set contains a point that is a limit point from the left of the set, while for every real $x$ there is a $y<x$ s.t. $(y,x)$ is a subset of $O_n$. The function $f$ is continuous at points of $B:=\cap O_n$, and the complemented of $B$ is countable.

$\endgroup$
1
  • $\begingroup$ Nice argument. It is simple and gives the optimal result. $\endgroup$ Nov 14, 2013 at 17:41
11
$\begingroup$

Any left-continuous function $f \colon \mathbb{R} \to \mathbb{R}$ is continuous almost everywhere.

To see this, consider for instance $f$ on $[0,1]$. (Note that left-continuous functions are Borel.) Take $\epsilon >0$ and for each $n \in \mathbb{N}$ a $\delta_n>0$ such that $$\mathcal{L}(A_n) \ge 1-2^{-n}\epsilon$$ with $$A_n := \left\{x \in [0,1]\,:\, |f(x)-f(y)|<\frac1n \text{ for all }y \in (x-\delta_n,x)\right\}.$$ Then $f$ is clearly continuous at the density-points of $\bigcap_{n \in \mathbb{N}} A_n$.

$\endgroup$
4
  • $\begingroup$ Can you explain why $A_n$ is measurable? Why do you mention "left continuous functions are Borel"? $\endgroup$
    – Hao Yin
    Nov 14, 2013 at 12:42
  • $\begingroup$ I did not think about the measurability too much, but you could consider for instance functions $F(x) = \sup_{\delta \in (0,\delta_n)\cap \mathbb{Q}}|f(x)-f(x-\delta)|$ which are measurable as countable supremum of measurable functions. $\endgroup$ Nov 14, 2013 at 13:00
  • $\begingroup$ As a nice consequence, if a left-continuous $f$ is bounded on $[a,b]$ it is also Riemann integrable. $\endgroup$ Nov 14, 2013 at 18:47
  • $\begingroup$ What is your $\mathcal{L}$? $\endgroup$
    – Bach
    Jan 21, 2020 at 0:55

Your Answer

By clicking “Post Your Answer”, you agree to our terms of service and acknowledge you have read our privacy policy.

Not the answer you're looking for? Browse other questions tagged or ask your own question.